En realidad es cierto, de una manera casi trivial. El teorema de Ehrenfest establece que, \begin{equation} \frac{d}{dt}\langle x\rangle=\langle p\rangle,\quad \frac{d}{dt}\langle p\rangle =- \langle V'(x)\rangle \end{equation} Sin embargo, para todas las funciones propias del oscilador armónico $\langle x\rangle=0$ (y por lo tanto $\langle V'(x)\rangle=0$ ) y $\langle p\rangle=0$ . Por tanto, el teorema de Ehrenfest sobre los estados propios se reduce a $0=0$ .
Se puede ver que la versión general del teorema de Ehrenfest funciona trivialmente para todos los estados propios. Afirma que para el observable arbitrario $A$ su valor de expectativa satisface la ecuación, \begin{equation} \frac{d}{dt} \langle A\rangle=\frac{1}{i\hbar}\langle [A,H]\rangle+\langle \frac{\partial A}{\partial t}\rangle \end{equation} Sin embargo, en los estados propios, \begin{equation} \langle\psi_n| [A,H]|\psi_n\rangle=\langle\psi_n|AH-HA|\psi_n\rangle=E_n\langle\psi_n|A-A|\psi_n\rangle=0 \end{equation} Así que el valor de expectativa del observable que no depende explícitamente del tiempo, no evoluciona en los estados propios, que es lo que cabría esperar.
Entonces, ¿a dónde conduce el teorema de Ehrenfest a la dinámica clásica? Hay que considerar los paquetes de ondas localizados. El ejemplo más sencillo sería el estado coherente del oscilador armónico que es el paquete de ondas gaussiano que sigue la trayectoria clásica
Para el oscilador armónico, el teorema de Ehrenfest es siempre "clásico", aunque sólo sea de forma trivial (como en el caso de los estados propios). Sin embargo, en general, el teorema de Ehrenfest se reduce a la ecuación clásica del movimiento sólo en tales paquetes de ondas localizadas que se concentran cerca de la trayectoria clásica como $\hbar$ llega a cero. El punto clave resulta ser el intercambio $\langle V'(x)\rangle \mapsto V'(\langle x\rangle)$ que en los estados generales no se puede hacer. Así que si quieres recuperar algo de dinámica clásica a partir de la teoría cuántica busca en los paquetes de ondas localizados.
0 votos
Tal vez esta pregunta mía pueda responder a su pregunta: physics.stackexchange.com/questions/267835/
1 votos
Aunque no responde a la pregunta planteada, solía hacer que los alumnos trazaran la distribución de probabilidad de posición para los casos cuántico y clásico en un intervalo de energías correspondiente a, digamos, $n = 1,2,3,4,10$ y un número "grande" como $20$ . Pondré una cifra en el chat: chat.stackexchange.com/transcript/message/49407141#49407141 .